Đến nội dung

Hình ảnh

$\sum \frac{a^{2}}{\sqrt{(1+a^{3})(1+b^{3})}}\geqslant \frac{4}{3}$


  • Please log in to reply
Chủ đề này có 1 trả lời

#1
buitudong1998

buitudong1998

    Trung úy

  • Thành viên
  • 873 Bài viết

Cho các số thực dương $a,b,c$ thỏa mãn: $abc=8$. CMR: $\sum \frac{a^{2}}{\sqrt{(1+a^{3})(1+b^{3})}}\geqslant \frac{4}{3}$


Đứng dậy và bước tiếp

#2
nk0kckungtjnh

nk0kckungtjnh

    Thượng sĩ

  • Thành viên
  • 254 Bài viết

Cho các số thực dương $a,b,c$ thỏa mãn: $abc=8$. CMR: $\sum \frac{a^{2}}{\sqrt{(1+a^{3})(1+b^{3})}}\geqslant \frac{4}{3}$

Lời giải: 

 Chú ý rằng với điểm rơi $a=b=c=3$ thì theo BĐT $AM-GM$: 

 

Ta có: $\sqrt{a^{3}+1}=\sqrt{(a^{2}-a+1)(a+1)} \leq \frac{a^{2}+2}{2}$. Tương tự thì $\sqrt{b^{3}+1} \leq \frac{b^{2}+2}{2}$

 

$\Rightarrow \frac{a^{2}}{\sqrt{(a^{3}+1)(b^{3}+1)}}\geq \frac{4a^{2}}{(a^{2}+2)(b^{2}+2)}$

 

Do đó ta cần chứng minh $\sum \frac{4a^{2}}{(a^{2}+2)(b^{2}+2)}\geq \frac{4}{3}$

 

Ta sẽ chứng minh BĐT sau là đúng: $ xy+yz+xz+2(x+y+z) \geq \frac{1}{3}(x+2)(y+2)(z+2)$ $(*)$

 

Trong đó $x=a^{2};y=b^{2};z=c^{2} \Rightarrow xyz=64$

 

Mặt khác ta dễ dàng chứng minh $(*)$ bằng BĐT $AM-GM$ như sau:

 

$(*) \Leftrightarrow xy+yz+xz+2(x+y+z) \geq 3\sqrt[3]{(xyz)^{2}}+2.3\sqrt[3]{xyz}=72$ (đúng)

 

Cực trị đạt được tại tâm 


Bài viết đã được chỉnh sửa nội dung bởi nk0kckungtjnh: 07-05-2014 - 21:19

             Hãy Đánh Bại Những Gì Yếu Đuối Để Biết Rằng


         Nỗ Lực Hơn Hẳn Tài Năng

- Nhân Chính -

 





1 người đang xem chủ đề

0 thành viên, 1 khách, 0 thành viên ẩn danh